LSAT Logic Games : Grouping Games

Study concepts, example questions & explanations for LSAT Logic Games

varsity tutors app store varsity tutors android store

Example Questions

Example Question #31 : Grouping Games

A dance teacher is creating a set list for an upcoming dance recital. There must be five numbers, performed in sequence. The numbers can be solos, duets or trios, made up of eight students: Amy, Belle, Carrie, Dana, Linda, Monique, Nicole and Ona. Each student will perform only once and the set list must conform to the follwing conditions:

 

Amy and Ona must perform solo

Carrie cannot perform in the second number unless Belle performs in the fourth

Nicole and Linda must perform together

Ona must perform at some time before Dana

The first and fifth numbers must be solos

If Dana is a member of a trio performing second, which of the following could be true?

Possible Answers:

Carrie performs second and Amy perfoms third

Ona performs third and Linda performs fourth

Ona performs first and Linda performs fifth

Belle performs first and Linda performs second

Carrie performs second and Nicole performs third

Correct answer:

Carrie performs second and Nicole performs third

Explanation:

As soon as we know that Dana performs second we can place Ona in the first spot. Therefore any answer that has someone other than Ona performing first OR has Ona performing somewhere other than first cannot be true. We know that the fifth spot must be a solo, so the answer that has Linda performing fifth must be wrong automatically, because she always has to perform with Nicole. If we place Carrie in the second spot with Dana we automatically place Belle in the fourth spot because of our conditional. If Amy performs third, we know she has to perform alone. This leaves us with Nicole, Linda and Monique left to place. Since Nicole and Linda have to perform together, Monique must perform fifth alone. We then have to place Nicole and Linda in the fourth spot with Belle, since Amy always performs alone and placing them in the second spot with Dana and Carrie would create a quartet. In this scenario, Dana is NOT in a trio, so this answer is incorrect. The correct answer (Carrie performs second and Nicole performs third) yields the following set list: First: Ona; Second: Dana, Carrie, Monique; Third: Nicole, Linda; Fourth: Belle; Fifth: Amy.

Example Question #32 : Grouping Games

A dance teacher is creating a set list for an upcoming dance recital. There must be five numbers, performed in sequence. The numbers can be solos, duets or trios, made up of eight students: Amy, Belle, Carrie, Dana, Linda, Monique, Nicole and Ona. Each student will perform only once and the set list must conform to the follwing conditions:

 

Amy and Ona must perform solo

Carrie cannot perform in the second number unless Belle performs in the fourth

Nicole and Linda must perform together

Ona must perform at some time before Dana

The first and fifth numbers must be solos

Which of the following must be true?

Possible Answers:

There are at most two duets

There is at most one trio

There are at least three duets

There are at least three solos

There are at most two solos

Correct answer:

There is at most one trio

Explanation:

We can figure this one out from the initial set-up of the game. If we are placing eight dancers into five numbers, we know there has to be one girl in each spot. This takes care of five dancers. We have three left over. Our options are to place all three in different numbers, yielding a 1:1:2:2:2 ratio OR to place two in one number and one in another, yielding a 1:1:1:2:3 ratio. Therefore the only correct answer is that there is at most one trio.

Example Question #33 : Grouping Games

The chairman of the board is creating a finance committee from a group of seven representatives: Harris, Innis, Jenkins, Kenzi, Lawrence, McHenry and Nin. The committe must conform to the following restrictions:

If Harris is chosen for the committee, Innis must also be chosen

If McHenry is chosen, Harris is not

If Nin is chosen, McHenry is also chosen but Innis is not

If Innis is chosen, Kenzi is also chosen

If Jenkins is not chosen, Harris is chosen

 

Which of the following is a possible complete and accurate list of representatives NOT chosen for the committee?

Possible Answers:

Nin, Jenkins, Lawrence

Harris, Innis, Jenkins, Kenzi, Lawrence

Jenkins, Nin

Jenkins, Lawrence, McHenry, Nin

Nin, McHenry, Kenzi

Correct answer:

Jenkins, Lawrence, McHenry, Nin

Explanation:

This is a simple list question turned inside out. In order to answer this question we need to figure out who is chosen for the committe in each answer and elminate answers based on rule violations. We know that Harris and McHenry cannot both be chosen and Nin and Innis cannot both be chosen. If Innis is chosen, Kenzi must also be chosen, as when Harris is chosen Innis must also be chosen. We also know that at least one of Harris and Jenkins must always be chosen, so any answer that has both of them out is wrong. The only correct possibility leaves this committee: Harris, Innis, Kenzi.

Example Question #541 : Lsat Logic Games

The chairman of the board is creating a finance committee from a group of seven representatives: Harris, Innis, Jenkins, Kenzi, Lawrence, McHenry and Nin. The committe must conform to the following restrictions:

If Harris is chosen for the committee, Innis must also be chosen

If McHenry is chosen, Harris is not

If Nin is chosen, McHenry is also chosen but Innis is not

If Innis is chosen, Kenzi is also chosen

If Jenkins is not chosen, Harris is chosen

 

If Harris is chosen for the committee, which of the following could be true?

Possible Answers:

Jenkins is chosen

Innis is not chosen

Kenzi is not chosen

McHenry is chosen

Nin is chosen

Correct answer:

Jenkins is chosen

Explanation:

We know that when Harris is chosen, Innis must also be chosen. If Innis is chosen, Kenzi must also be chosen. So the smallest possible group is Harris, Innis and Kenzi. Nin and Innis cannot both be chosen, so Nin cannot be in this group. McHenry and Harris cannot both be chosen, so McHenry also cannot be in this group. The only answer that could be true is that Jenkins is chosen - Jenkins does not have to be in this group but could be.

Example Question #542 : Lsat Logic Games

The chairman of the board is creating a finance committee from a group of seven representatives: Harris, Innis, Jenkins, Kenzi, Lawrence, McHenry and Nin. The committe must conform to the following restrictions:

If Harris is chosen for the committee, Innis must also be chosen

If McHenry is chosen, Harris is not

If Nin is chosen, McHenry is also chosen but Innis is not

If Innis is chosen, Kenzi is also chosen

If Jenkins is not chosen, Harris is chosen

What is the smallest possible number of representatives chosen for the committee?

Possible Answers:

Zero

One

Four

Three

Two

Correct answer:

One

Explanation:

We know that if Jenkins is not chosen, Harris is. Therefore if Harris is not chosen, Jenkins is. This means that we must always have at least one or the other of these two in the game. If Harris is chosen this leads to several other members being chosen as well, but if Jenkins is chosen he could be the only representative on the committee.

Example Question #543 : Lsat Logic Games

The chairman of the board is creating a finance committee from a group of seven representatives: Harris, Innis, Jenkins, Kenzi, Lawrence, McHenry and Nin. The committe must conform to the following restrictions:

If Harris is chosen for the committee, Innis must also be chosen

If McHenry is chosen, Harris is not

If Nin is chosen, McHenry is also chosen but Innis is not

If Innis is chosen, Kenzi is also chosen

If Jenkins is not chosen, Harris is chosen

If there are excatly three people on the committee, each of the following could be true EXCEPT:

Possible Answers:

McHenry is chosen and Lawrence is chosen

Harris is chosen and Kenzi is chosen

Nin is chosen and Jenkins is chosen

Harris is chosen and Jenkins is chosen

Kenzi is chosen and Lawrence is chosen

Correct answer:

Harris is chosen and Jenkins is chosen

Explanation:

The only answer that would exceed three people is Harris and Jenkins are both chosen. When Harris is chosen we must also choose Innis and then Kenzi. Choosing Jenkins as well would require four people to be on the committee. The other answers yield the following groups: Harris, Kenzi, Innis; Kenzi, Lawrence, Jenkins; Nin, McHenry, Jenkins; McHenry, Lawrence, Jenkins

Example Question #544 : Lsat Logic Games

The chairman of the board is creating a finance committee from a group of seven representatives: Harris, Innis, Jenkins, Kenzi, Lawrence, McHenry and Nin. The committe must conform to the following restrictions:

If Harris is chosen for the committee, Innis must also be chosen

If McHenry is chosen, Harris is not

If Nin is chosen, McHenry is also chosen but Innis is not

If Innis is chosen, Kenzi is also chosen

If Jenkins is not chosen, Harris is chosen

All of the following must be true EXCEPT:

Possible Answers:

McHenry and Harris cannot both be chosen

Nin and Innis cannot both be chosen

McHenry and Innis can both be chosen

Harris and Jenkins cannot both be chosen

Nin and Harris cannot both be chosen

Correct answer:

Harris and Jenkins cannot both be chosen

Explanation:

The only false statement here is that Harris and Jenkins cannot both be chosen - no rule states this. We know from the rules that McHenry and Harris cannot both be chosen, and that Nin and Innis cannot both be chosen. Since McHenry must be chosen when Nin is chosen, Nin therefore cannot be chosen with Harris either. There is no rule that states that McHenry and Innis cannot be chosen together.

Example Question #545 : Lsat Logic Games

The chairman of the board is creating a finance committee from a group of seven representatives: Harris, Innis, Jenkins, Kenzi, Lawrence, McHenry and Nin. The committe must conform to the following restrictions:

If Harris is chosen for the committee, Innis must also be chosen

If McHenry is chosen, Harris is not

If Nin is chosen, McHenry is also chosen but Innis is not

If Innis is chosen, Kenzi is also chosen

If Jenkins is not chosen, Harris is chosen

 

If Nin is chosen, which of the following is a complete and accurate list of all other possible representatives who could also be chosen?

Possible Answers:

McHenry, Jenkins, Kenzi

McHenry, Jenkins

McHenry

McHenry, Jenkins, Lawrence

McHenry, Jenkins, Kenzi, Lawrence

Correct answer:

McHenry, Jenkins, Kenzi, Lawrence

Explanation:

To answer this question we start by choosing Nin. We then automatically choose McHenry and eliminate Innis. Because we have chosen McHenry, we eliminate Harris. Eliminating Harris forces us to choose Jenkins. All that are left are Kenzi and Lawrence, who could or could not be chosen. Therefore, if we wanted to create the largest possibly committee starting with Nin, we could add in McHenry, Jenkins, Lawrence and Kenzi. 

Example Question #546 : Lsat Logic Games

Four children and four adults attend a baseball game and sit in two sections. At least three members of the group must sit in each section--prime seating and the bleachers.  The adults were Matthew, Nora, Olga, and Peter.  The children were Sara, Tania, Ulric, and Victor. The following rules apply:

There must be at least one adult in each section

Nora and Sara cannot sit in the same section

Nora and Ulric must sit in the same section

Matthew must sit in the prime section

Which of the following could be a list of who sat in each section?

Possible Answers:

Prime: Ulric, Nora, Tania, Matthew, Victor, Peter

Bleachers: Sara, Olga

 

Prime: Ulric, Matthew, Olga, Sara

Bleachers: Peter, Nora, Victor, Tania

Prime: Matthew, Ulric, Nora

Bleachers: Peter, Sara, Victor, Tania, Olga

Prime: Ulric, Olga, Nora, Tania

Bleachers: Peter, Sara, Victor, Matthew



Prime: Peter, Nora, Ulric, Matthew, Olga

Bleachers: Tania, Sara, Victor

Correct answer:

Prime: Matthew, Ulric, Nora

Bleachers: Peter, Sara, Victor, Tania, Olga

Explanation:

Make sure you accurately understand each rule, then apply the rules to eliminate each incorrect answer.  Each incorrect answer directly violates at least one rule.

Example Question #547 : Lsat Logic Games

Four children and four adults attend a baseball game and sit in two sections. At least three members of the group must sit in each section--prime seating and the bleachers.  The adults were Matthew, Nora, Olga, and Peter.  The children were Sara, Tania, Ulric, and Victor. The following rules apply:

There must be at least one adult in each section

Nora and Sara cannot sit in the same section

Nora and Ulric must sit in the same section

Matthew must sit in the prime section

Which of the following two people could not sit together?

Possible Answers:

Nora and Ulric

Sara and Ulric

Victor and Tania

Victor and Sara

Matthew and Nora

Correct answer:

Sara and Ulric

Explanation:

Sara cannot be in the same section as Nora, but Nora must be in the same section as Ulric.  As a result, Sara is prohibited from being in the same section as Ulric, just as she is prohibited from being in the same section as Nora.

Tired of practice problems?

Try live online LSAT prep today.

1-on-1 Tutoring
Live Online Class
1-on-1 + Class
Learning Tools by Varsity Tutors